The function below models the correlation
between the number of hours a plant is kept in
sunlight (x) and the height (y), in mm, to which it
grows:
y = 2 + 4x
What does the y-intercept of this function
represent?
The original height of the plant was 4 mm.
The original height of the plant was 2 mm.
The height of the plant increases by 2 mm for
every hour of sunlight it receives.
The height of the plant increases by 4 mm for
every hour of sunlight it receives.

Answers

Answer 1
the original height was 4 mm

Related Questions

The curved sides of large storage tanks at a refinery need to be painted. What is the approximate area of each tank that will be painted?



OA. 9,500 ft²
OB. 1,144 ft²
OC. 4,574 ft²
OD. 2,287 ft²

Answers

Correct Answer is D. 2,287 ft

Find the probability. Leave your answer in simplest fraction form.
You roll a six-sided die. The die shows an even number or a number
greater than three.

Answers

Answer:

5/6

Step-by-step explanation:

The numbers on a six-sided die are as shown:

1, 2, 3, 4, 5, 6

The even numbers are:

2, 4, 6

The numbers greater than 3 are:

3, 4, 5, 6

Both lists together are:

2, 3, 4, 5, 6

Because 5 out of 6 numbers satisfy these conditions, the probability of satisfying these conditions is 5/6.

The answer to this is 5,6

what is high common factor

Answers

Answer:

The highest number that divides each of the two or more numbers is the HCF or Highest Common Factor.

Step-by-step explanation:

Example: What is the HCF of {12,42,14}

Answer: The HCF is 2 as the biggest number they can all be divided by is 2. {12,42,14} = {2^2*3,2*3*7,2*7} The factors in each number is 2.

For example if your trying to find the highest common factor of 8 and 12 it has to be less or equal to 12 will the answer is 4 because it is able to be divided into any without any reminder. Four has both eight and twelve when you times it by 2 or 3

4,8,12,16

The following are the ages (years) of 5 people in a room:

13
,


12
,


13
,


15
,


12

A person enters the room.
The mean age of the 6 people is now 20.
What is the age of the person who entered the room?

Answers

Answer:

55

Step-by-step explanation:

To find the average you add up all numbers given then divide by the amount of numbers there were. The mean, or average, of the original 5 people was 13 since their ages added was 65, which you divide by 5 for 13. The new mean once the 6th person entered the room was 20. So, you do 6 [number of people] times 20 [average] to get 120. Taking 65 [added amount of original ages of the 5 people] away from 120 [the new added up ages of all 6 people], you get 55. This means 55 would be the 6th person's age.

Sorry if the explanation is confusing!

Call the total ages of the 6 people, x

To calculate the mean age of 6 people, we take the total age (x) and divide by the amount of people (6)

So, x : 6 = 20

-> The total ages of the 6 people are 20 x 6 = 120

We can calculate the 6th person's age by taking 120 and minus the total age of the other 5

-> 120 - (13+12+13+15+12) = 55

So, the 6th person's age is 55.

Under his cell phone plan, Elijah pays a flat cost of $46.50 per month and $4 per
gigabyte. He wants to keep his bill at $58.50 per month. How many gigabytes of data
can he use while staying within his budget?

Answers

Answer:

3

Step-by-step explanation:

Start with the amount that he wants to pay with gigabytes and subtract the amount of his bill without gigabytes.

58.50-46.50=12

 Then divide that amount by the price of each gigabyte to determine how many gigabytes he can use while staying in budget.

12/4=3

Answer: 3

Inequalities help us to compare two unequal expressions. The gigabytes of data that Elijah should use while staying within his budget is 3 gigabytes.

What are inequalities?

Inequalities help us to compare two unequal expressions. Also, it helps us to compare the non-equal expressions so that an equation can be formed. It is mostly denoted by the symbol <, >, ≤, and ≥.

Given that Elijah pays a flat cost of $46.50 per month and $4 per gigabyte. He wants to keep his bill at $58.50 per month. Therefore, we can write the inequality for this condition as,

$46.50 + $4(x) ≤ $58.50

46.50 + 4x ≤ 58.50

4x ≤ 58.50 - 46.50

4x ≤ 12

x ≤ 12/4

x ≤ 3

Hence, the gigabytes of data that Elijah should use while staying within his budget is 3 gigabytes.

Learn more about Inequality here:

https://brainly.com/question/19491153

#SPJ2

seperate 90 into two parts so that one part is four times the other

Answers

Answer:

18 and 72

Step-by-step explanation:

the smaller part can be assigned as x while the larger will be 4x. Both numbers need to add up to 90, giving the equation: 4x+x = 90

Solve:

4x+x = 90

5x = 90

x = 18

4x = 72

aimee has 20 apples and wants to have the same amount of apples in each bag but ends up with more than one bag of apples and each bag to contain more than one apple
work out all the possibilities for all of the number of apples that she could use for each bag

Answers

Answer:

There are 4 possibilities: 2, 4, 5 or 10 apples in each bag.

===========

Let the number of apples in each bag is x, then the number of bags is 20/x.

Since the number of apples in each bag is greater than one and number of bags is greater than one, we have the following conditions:

x > 1 , 20/x > 1 ⇒ x < 20,x is factor of 20.

The factors of 20 between 1 and 20 are:

2, 4, 5, 10

The possibilities are:

If x = 2, then the number of bags is 20/2 = 10;If x = 4, then the number of bags is 20/4 = 5;If x = 5, then the number of bags is 20/5 = 4;If x = 10, then the number of bags is 20/10 = 2.

If an artist combines 6 L of 33% by volume solution and 4 L of 60% by volume solution, what's the percent by volume of the resulting solution

Answers

The percent by volume of the resulting solution is 10L by 93 percent

How to determine the volume

The resulting solution = sum of the volumes of both solutions

Resulting solution = 6L × 33 percent + 4L × 60 percent

Resulting solution = 10 L × 93 percent

This is so because both solutions add up to form the resulting solution

Thus, the percent by volume of the resulting solution is 10L by 93 percent

Learn more about equivalent ratio here:

https://brainly.com/question/2328454

#SPJ1

Please answer this question fast (Lines and angles ch...class9)

Answers

The angle of y in the triangle is 30 degrees.

How to find angles in a triangle?

∠O = 180 - 30 - 70 (angles in a triangle)

∠O = 180 - 100(angles in a triangle)

∠O = 80°

Using vertically opposite angle principle and sum of angle in a triangle,

80 + y + 70 = 180

180 - 150 = y

y = 30 degrees

learn more on angles here: https://brainly.com/question/17738167

#SPJ1

A study on the latest fad diet claimed that the amounts of weight lost by all people on this diet had a mean of 23.4 pounds and a standard deviation of 6.8 pounds.
Step 2 of 2 : If a sampling distribution is created using samples of the amounts of weight lost by 63 people on this diet, what would be the standard deviation of the sampling distribution of sample means? Round to two decimal places, if necessary.

Answers

Using the Central Limit Theorem, the standard deviation of the sampling distribution of sample means would be of 0.86.

What does the Central Limit Theorem state?

It states that the standard deviation of the sampling distribution of sample means is given by:

[tex]s = \frac{\sigma}{\sqrt{n}}[/tex]

In which:

[tex]\sigma[/tex] is the standard deviation of the population.n is the sample size.

The parameters for this problem are given as follows:

[tex]\sigma = 6.8, n = 63[/tex].

Hence:

[tex]s = \frac{\sigma}{\sqrt{n}}[/tex]

[tex]s = \frac{6.8}{\sqrt{63}}[/tex]

s = 0.86.

More can be learned about the Central Limit Theorem at https://brainly.com/question/16695444

#SPJ1

give the meaning of each expression.
16^5
y^4
7a^3

Answers

Answer:

16 to the power of 5

y to the power of 4

(7a) to the power of 3 or 7 times a to the power of 3

For which value of k is the value of k(k-2)(k+1) negative

Answers

Answer:

[tex](-\infty, -1)\ \cup\ (0, 2)[/tex]

Step-by-step explanation:

So if you expand out the two binomials (k-2)(k+1), you'll get: [tex]k^2+k-2k-2[/tex]. which simplifies to: [tex]k^2-k-2[/tex]. Multiplying this by the k gives you: [tex]k^3-k^2-2k[/tex]. As you can see the degree is odd, this means that this polynomial will have two opposite end behaviors. And as you can see the leading coefficient is positive, meaning that this function will go towards positive infinity as k goes towards positive infinity. Also we if look at the original equation given, it's in factored form, with the zeroes as k=0, k=2, and k=-1. So given this we can draw a simple graph to see when the value of the equation is negative. If you look at the graph I drew you'll see that it's negative from (-infinity, -1) and then negative from (0, 2)

how many real solutions does this system of equations have?
y=x2+1
y=x

A. 1
B. 2
C. 3
D. 0

Answers

The system of equations has (d) 0 real solutions

How to determine the number of real solutions?

The system of equations is given as:

y = x^2+1

y=x

Substitute y=x in y = x^2+1

x = x^2+1

This gives

x^2 - x + 1 = 0

Calculate the discriminant using:

d = b^2 - 4ac

So, we have:

d = (-1)^2 - 4 * 1 * 1

Evaluate

d = -3

Because the discriminant is negative, the equation has no real solution

Hence, the system of equations has (d) 0 real solutions

Read more about system of equations at:

https://brainly.com/question/12895249

#SPJ1

Which statement regarding the diagram is true W, X,Z,Y

Answers

The statement that's true about the triangle is that WXY + YXZ = 180°.

How to illustrate the information?

It should be noted that in the question, the options are related to a linear pair.

According to the linear pair, when a line cuts another line at a point, then the sum of the adjacent angles formed at the point will be 180°.

Therefore, WXY + YXZ = 180°.

The correct option is C.

The complete question is:

Which statement regarding the diagram is true?

m∠WXY = m∠YXZ

m∠WXY < m∠YZX

m∠WXY + m∠YXZ = 180°

m∠WXY + m∠XYZ = 180°

Learn more about triangles on:

brainly.com/question/4001545

#SPJ1

Area=
Help me please!! Thank u so much

Answers

The area of the triangle is 78 units squared. Why? Well, the formula for calculating the area of a triangle is 1/2bh, which simply means to multiply base times height times 1/2. You could also multiply base times height then divide your answer by 2, because that’s the same thing as multiplying by 1/2.

Our base of the triangle: 13 units

Our height of the triangle: 12 units

Because the formula requires multiplying base times height, we can do 13 x 12. 13 x 12 = 156. However, we are not done. We now must multiply our answer (156) by 1/2 or divide it by 2. I would recommend to divide it by 2 because it is easier that way. 156 divided by 2 gives us 78. Therefore, by using the formula bh/2, we will find that our result is 78 units squared. If you need to better understand this, let me know and I will gladly assist you.

Answer:

area = 78

Step-by-step explanation:

Area of triangle = 1/2  x  base  x  perpendicular height

                            ⇒ 1/2  x  13  x  12

                            ⇒ 78 units²

Determine if each table below represents a linear function, quadratic function, or neither.

Answers

The table below is a quadratic function.

What is a quadratic function?

A quadratic function is an Algebraic function with the power of its variable as 2.

This function normally has three algebraic terms, the x-square term, the x-term and the constant term.

Analysis:

The X-column values increases from -2 to 1, while the f(x) column which is the y-column increase from -9, until it gets to 1, where the value falls to -5.

A typical behavior of an inverted-v curve which is a quadratic curve.

If it were a linear function, as x values increase, y value would either keep increasing or decreasing, it does not change its orientation.

Learn more about quadratic curves: brainly.com/question/1214333

#SPJ1

URGENT!!!WORTH 27 POINTS!!!

Doug can download new songs for $1.19 each. Write an equation to show how many songs he can download for $12.00.

A. 12x=1.19
B. 12+x=1.19
C. 1.19+x=12
D. 1.19x=12

Answers

Answer:

1.19x = 12

Step-by-step explanation:

To write an equation, we multiply the cost of each song by the number of songs, x

1.19 * x

This is equal to the total amount he is allowed to spend

1.19x = 12

Answer:

D

Step-by-step explanation:

If Doug can download new songs for $1.19 each, then he will be able to download 12 songs.

The 8 foot diameter circular table has a 4 foot wide extension. What is the total area with the extension? How does the area compare to the area of the 10 foot diameter table? Show your work. (3 points)

Answers

Answer: the total area with the extension S≈82,3 foot²,  S>S'.

Step-by-step explanation:

D₁=8 foot    D₂=10 foot    a wide extension = 4 foot.

1) Let the total area with the extension S is the area of the circular table S₁

plus a wide extension S₂.

Considere S₁:

[tex]R_1=\frac{D_1}{2} \\R_1=\frac{8}{2} \\R_1=4 foot.\\S_1=\pi* R_1^2\\S_1=\pi *4^2\\S_1=16*\pi \\S_1\approx50,3\ foot^2.\\[/tex]

[tex]S_2=8*4\\S_2=32 \ foot^2.[/tex]

[tex]S\approx50,3+32\\S\approx82,3 \ foot^2.[/tex]

2)\ Considere S':

[tex]R_2=\frac{D_2}{2} \\R_2=\frac{10}{2} \\R_2=5 \ foot.\\S'=\pi *R^2\\S'=\pi *5^2\\S'=25*\pi \\S'\approx78,5\ foot^2.[/tex]

S>S'.

Good luck an' have a nice day!

X = square root=
Help me please thanks

Answers

The value of x will be equal to 33.94 units.

What is trigonometry?

Trigonometry is the branch of mathematics that set up a relationship between the sides and angle of the right-angle triangles.

Given that:-

Given that the radius of the circle is 12 units which are PO = 12 and OR = 12, QR = x , ∠O = 60 So ∠Q = 30.

First, we will calculate the Length OQ by angle property in triangle OQR.

[tex]Sin 30=\dfrac{Perpendicular}{Hypotenuse}[/tex]

[tex]Sin 30 = \dfrac{12}{OQ-12}[/tex]

[tex]\dfrac{1}{2}=\dfrac{12}{OQ-12}[/tex]

OQ - 12 = 24

OQ = 36

Now applying the Pythagorean theorem in the triangle OQR.

H² = P² + B²

36² = 12² + x²

x² = 1296 - 144

x = √1152

x = 33.94

Therefore the value of  x will be equal to 33.94 units.

To know more about Trigonometry follow

https://brainly.com/question/24349828

#SPJ1

There are two numbers. One number is twice the other number. The difference of the smaller number and half the larger number is 20.

An equation created to find the smaller number will have

Answers

Step-by-step explanation:

There cannot be such a question because the difference of half of the big number and the small number will automatically be zero, but we can find it with the equation I circled on the paper I gave you. achievements

Class A has 13 pupils and class B has 16 pupils.
Both classes sit the same maths test.
The mean score for class A is 20.
The mean score for class B is 45.
What is the mean score (rounded to 2 DP) in the maths test across both classes?

Answers

Answer:

ok so we can just add 20 + 45 then divide by 2

20+45=65/2=32.5

32.5

Hope This Helps!!!

Which function has zeros at x = -2 and x = 5?
O f(x) = x2 + 2x - 10
• f(x) = x2 - 2x - 10
O f(x) = X2 + 3x - 10
® f(x) = x2 - 3x - 10

Answers

Answer:

f(x)=x²-3x-10

Step-by-step explanation:

[tex]f(x) = x {}^{2} - 3x - 10 \\ to \: find \: x \: intercept \:o r \: zero \: substitute \: f(x) = 0\: \\ 0 = x {}^{2} - 3x - 10 \\ x {}^{2} - 3x - 10 = 0 \\ x {}^{2} + 2x - 5x - 10 = 0 \\ x(x + 2) - 5x - 10 = 0 \\ x(x + 2) - 5(x + 2) = 0 \\ (x + 2).(x - 5) = 0 \\ x + 2 = 0 \\ x - 5 = 0 \\ x = - 2 \\ x = 5[/tex]

therefore the zeros of the equation are x₁=-2,x₂=5

(2/5) power of 2 as a fraction.


Please help.

khan academy.

Answers

[tex]\Large\maltese\underline{\textsf{Our problem:}}[/tex]

What is [tex]\bf{\bigg(\dfrac{2}{5}\bigg)^2[/tex]?

[tex]\Large\maltese\underline{\textsf{This problem has been solved!}}[/tex]

When we have a fraction that is raised to a specific power, we raise both the numerator and the denominator to that power.

[tex]\bf{\dfrac{2^2}{5^2}}=\dfrac{4}{25}[/tex]

[tex]\rule{300}{1.7}[/tex]

[tex]\bf{Answer:}[/tex]

                    [tex]\bf{=\dfrac{4}{25}[/tex]

[tex]\boxed{\bf{aesthetic \not101}}[/tex]

Using mod, find the remainder of 3^51 when divided by 7. Please show steps on how to use modulu, am a bit confused.

Answers

Answer:

[tex]6[/tex]

Step-by-step explanation:

The gist of modular arithmetic in a nutshell: the numbers [tex]a[/tex] and [tex]b[/tex] are considered to be congruent by their modulus [tex]m[/tex] if [tex]m[/tex] is a divisor of their difference.

In mathematics: [tex]a \equiv_{m} b \Leftrightarrow (a - b) \vdots m[/tex]

Exemplifying this: [tex]6 \equiv_{7} -1[/tex] because [tex]6 - (-1) = 6 + 1 = 7[/tex], [tex]7 \vdots 7[/tex].

Let us have following equivalences: [tex]a \equiv_{m} b[/tex] and [tex]c \equiv_{m} d[/tex], then:  [tex](a - b) \vdots m[/tex] and [tex](c - d) \vdots m[/tex] by definition.

Properties:

1. [tex]a + c \equiv_{m} b + d \Leftrightarrow ((a + c) - (b + d)) \vdots m \Leftrightarrow (a + c - b - d) \vdots m \Leftrightarrow ((a - b) + (c - d)) \vdots m[/tex].

2. [tex]a - c \equiv_{m} b - d \Leftrightarrow ((a - c) - (b - d)) \vdots m \Leftrightarrow (a - c - b + d) \vdots m \Leftrightarrow ((a - b) - (c - d)) \vdots m[/tex].

3. [tex]ac \equiv_{m} bd \Leftrightarrow (ac - bd) \vdots m \Leftrightarrow (ac - bc - bd + bc) \vdots m \Leftrightarrow (c(a - b) + b(c - d)) \vdots m[/tex].

4. What if we have [tex]a \equiv_{m} b[/tex] twice? If we abide by property 3, we can come to the conclusion that [tex]a^2 \equiv_m b^2[/tex]. It is fair enough that there is room for the equivalence [tex]a^n \equiv_{m} b^n[/tex].

[tex]3^{51} = (3^3)^\frac{51}{3} = 27^{17} \equiv_{7} (-1)^{17} \equiv_{7} -1 \equiv_{7} 6[/tex].

We used property 4.

Keep in mind that any remainder cannot be a negative number.

Therefore, the remainder equals [tex]6[/tex].

Deepak wrote out the steps to his solution of the equation startfraction 5 over 2 minus 3 x minus 5 plus 4 x equals negative startfraction 7 over 4 endfraction – 3x – 5 4x = –.

Answers

The solution is x=3/4

How can we solve given equation?

First, we will solve like terms. Then shift constant to other side and keep x on the same side to get the value of x.

We can solve given equation as shown below:

5/2-3x-5+4x=-7/4

(5-10)/2+x=-7/4

-5/2+x=-7/4

x=5/2-7/4

x= (10-7)/4

x=3/4

Hence, the solution is x=3/4.

Learn more about Linear equations here:

https://brainly.com/question/43297

#SPJ4

Ab and cd with m as the midpoint of both ab and cd. ab = 6.4 cm and cd = 4.0 cm. a, b and c are not collinear.

Answers

From the straight line AB and CD with point M as midpoint of both lines, AM = BM = 3.2 cm and CM = DM = 2 cm

What is an equation?

An equation is an expression that shows the relationship between two or more variables and numbers.

m as the midpoint of both ab and cd, hence:

AM = BM = AB/2 = 6.4/2 = 3.2 cm

CM = DM = CD/2 = 4/2 = 2 cm

From the straight line AB and CD with point M as midpoint of both lines, AM = BM = 3.2 cm and CM = DM = 2 cm

Find out more on equation at: https://brainly.com/question/13763238

#SPJ1

Using the following image, solve for SR. Look at the image closely.
What would SR be? Giving Brainly!

Answers

Step-by-step explanation:

I assume

SR = 2x + 23

RQ = x + 21

if that is true, then the situation is completely simple :

14 = (2x + 23) + (x + 21) = 3x + 44

3x = -30

x = -10

SR = 2×-10 + 23 = -20 + 23 = 3

RQ = -10 + 21 = 11

[tex]\quad \huge \quad \quad \boxed{ \tt \:Answer }[/tex]

[tex]\qquad \tt \rightarrow \:x = -10 [/tex]

____________________________________

[tex] \large \tt Solution \: : [/tex]

[tex] \qquad \tt \rightarrow \: SR + RQ = SQ[/tex]

[tex]\qquad \tt \rightarrow \: 2x + 23 + x + 21 = 14[/tex]

[tex]\qquad \tt \rightarrow \: 3x + 44 = 14[/tex]

[tex]\qquad \tt \rightarrow \: 3x = 14 - 44[/tex]

[tex]\qquad \tt \rightarrow \: 3x = - 30[/tex]

[tex]\qquad \tt \rightarrow \: x = - 10[/tex]

Now,

[tex] \qquad \tt \rightarrow \: SR = 2x + 23 [/tex]

[tex] \qquad \tt \rightarrow \: SR = 2(-10) + 23 [/tex]

[tex] \qquad \tt \rightarrow \: SR =-20+ 23 [/tex]

[tex] \qquad \tt \rightarrow \: SR = 3 \:\: units [/tex]

Answered by : ❝ AǫᴜᴀWɪᴢ ❞

Marta believes that the equation of the line of best fit for the scatterplot below is -
. Which statement best summarizes why Marta is likely incorrect?

Answers

Marta’s equation has a positive y-intercept, but the scatterplot shows a negative correlation.

What is a scatter plot?

The scatter plot is a manner in which data is presented as dots on a cartesian axes, The line of best fit is a description of the data that is presented in the scatter plot.

Hence, Marta is incorrect because Marta’s equation has a positive y-intercept, but the scatterplot shows a negative correlation.

Missing parts;

Marta believes that the equation of the line of best fit for the scatterplot below is y=-5/9x+23/9. Which statement best summarizes why Marta is likely incorrect?

Marta’s equation has a positive y-intercept, but the scatterplot suggests a negative y-intercept.

Marta’s equation has a positive y-intercept, but the scatterplot shows a negative correlation.

Marta’s equation has a negative slope, but the scatterplot suggests a negative y-intercept.

Marta’s equation has a negative slope, but the scatterplot shows a positive correlation.

Learn more about scatter plot:https://brainly.com/question/13984412

#SPJ1

The figures below are similar.
12 yd
30 yd
What are a) the ratio of the perimeters and b) the ratio of the areas of the larger figure to the
smaller figure? The figures are not drawn to scale.

Answers

Answer:

  a) 2.5

  b) 6.25

Step-by-step explanation:

For similar figures, the ratio of any corresponding linear dimensions is the same. The ratio of areas is the square of that.

Application

The ratio of linear dimensions, larger to smaller, is ...

  (30 yd)/(12 yd) = 2.5

a) Perimeter

Perimeter is a linear dimension, the sum of side lengths. The ratio of perimeters is 2.5.

b) Area

The ratio of areas, larger to smaller, is the square of the scale factor for side lengths:

  (2.5)² = 6.25

The ratio of the areas of the larger to smaller figure is 6.25.

Please help! Consider the sequence {20, 17, 14, 11, 8, 5, 2...}.

Answers

Answer:

c.) 38

Step-by-step explanation:

[tex]\sum\limits_{n=3}^6 a_n[/tex]   means "summation of the all the elements starting from the 3rd element to the 6th element".

The 3rd element in the series is 14, and the 6th element is 5; we have to add these and all the elements between them together.

∴    [tex]\sum\limits_{n=3}^6 a_n[/tex]    =  14 + 11 + 8 + 5

                   = 38

Other Questions
A gardener is planning to fill her garden with mulch. she plots it on a grid to plan how much she will need. the garden is in the shape of a rectangle with vertices at (3, 9) (5, 9) (3, 3) (5, 3). one bag of mulch covers 5ft^2 and costs $5.00. how much will it cost her to cover her garden? True or False. Recovery is possible without the active involvement of the person receiving services. Which quantity may be calculated directly using Newton's second law of motion?weightvelocitypositiondirection Explain the different techniques that can be used to create texture in a piece of art. 3. How do you think reading about internal conflicts in stories can be helpful to us inour own lives? Ramone is writing a paper about possible treatments for learning disorders. Which of these sources would give him the most useful information?A. an interview with someone who has a learning disorderB. a Web site that gives descriptions of every learning disorderC. an encyclopedia article on what causes learning disordersD. a medical journal about learning disorder research they are visiting phokhara simple past Pete drive 50 miles in 4 hours work out his average speed in miles per hour A _____________ is a cost-effective instance type that can be used for jobs that can be interrupted and resumed at any time. The eight wastes of traditional operations include all of the following except Question content area bottom Part 1 A. defects. B. overproduction. C. waiting. D. utilizing people to their full potential. Based on the elements in glucose, and the way the atoms are attached, you could determine thatglucose is an example ofcarbo molecule hydrogen molecule organic compound inorganic compound Let g(x) = f(x+3) for some function f(x). Is g(x) a function? From the previous problem, let the domain of f(x) be [1, 4). What is the domain of g(x)? An approximate solution to an equation is found using this iterative process.(x)-1and x = -14Xn+1 =4/34a) (i) Work out the value of x(ii) Work out the value of x3b) Work out the solution to 6 decimal places. How much electrical energy is involved in this transformation? 60 j 80 j 100 j 120 j 1. How could you figure out the coordinates of a reflection over the x-axis? What would the coordinates of that reflected point be?2. Then also tell how you could determine the coordinates when (-8,2) is reflected over the y-axis, and tell the coordinates of that new point. Writing Exercises587. When you convert a number from decimal notation to scientific notation, how do you know if the exponent will be positive or negative? John currently pays $500 monthly towards his minimum payments on his credit cards. After enrolling with National Debt Relief, John now pays $350 towards the same credit cards through his debt relief program. How much does John save off his minimums after 3 months? A triangle is shown. One side has a length of 16, another side has a length of 20, and the length of the longest side is unknown.Janice examines the given triangle and estimates that the longest side has a length of 25 unitsif it is a right triangle. How does her estimate compare to the actual length?It is exactly correct.It is under by approximately 0.6 units.It is over by approximately 0.6 units.It is over by 13 units. If the President of the United States meets with the Prime Minister of Great Britain to finalize negotiations of a treaty allowing the United States to use British territory to establish a military base, he is performing his role as __________. Which properties are necessary to claim that the two prisms are congruent? Check all that apply.